Polynomial of the nth degree and its elements.












2












$begingroup$


I have to prove, that polynomial:



$ x^n + a_{n-1}x^{n-1}+...+a_2x^2+a_1x+a_0=0$
, where $ a_i= [-1,0,1] $ and $ i=0,1,.....,n $

doesn't have any solutions in the set
$$
begin{aligned}
( -infty,-2) cup (2, infty)
end{aligned}
$$

We know, that $ a_n=1$, so we know, that $ x_1+x_2+....+x_n=-2$.
It's all I have though, any
suggestions, what can I do in next step?










share|cite|improve this question











$endgroup$








  • 1




    $begingroup$
    elcome to MSE. Your question is phrased as an isolated problem, without any further information or context. This does not match many users' quality standards, so it may attract downvotes, or be put on hold. To prevent that, please edit the question. This will help you recognise and resolve the issues. Concretely: please provide context, and include your work and thoughts on the problem. These changes can help in formulating more appropriate answers.
    $endgroup$
    – José Carlos Santos
    Dec 2 '18 at 8:31










  • $begingroup$
    "elcome?" 😁😁😁
    $endgroup$
    – Robert Lewis
    Dec 2 '18 at 22:44
















2












$begingroup$


I have to prove, that polynomial:



$ x^n + a_{n-1}x^{n-1}+...+a_2x^2+a_1x+a_0=0$
, where $ a_i= [-1,0,1] $ and $ i=0,1,.....,n $

doesn't have any solutions in the set
$$
begin{aligned}
( -infty,-2) cup (2, infty)
end{aligned}
$$

We know, that $ a_n=1$, so we know, that $ x_1+x_2+....+x_n=-2$.
It's all I have though, any
suggestions, what can I do in next step?










share|cite|improve this question











$endgroup$








  • 1




    $begingroup$
    elcome to MSE. Your question is phrased as an isolated problem, without any further information or context. This does not match many users' quality standards, so it may attract downvotes, or be put on hold. To prevent that, please edit the question. This will help you recognise and resolve the issues. Concretely: please provide context, and include your work and thoughts on the problem. These changes can help in formulating more appropriate answers.
    $endgroup$
    – José Carlos Santos
    Dec 2 '18 at 8:31










  • $begingroup$
    "elcome?" 😁😁😁
    $endgroup$
    – Robert Lewis
    Dec 2 '18 at 22:44














2












2








2


2



$begingroup$


I have to prove, that polynomial:



$ x^n + a_{n-1}x^{n-1}+...+a_2x^2+a_1x+a_0=0$
, where $ a_i= [-1,0,1] $ and $ i=0,1,.....,n $

doesn't have any solutions in the set
$$
begin{aligned}
( -infty,-2) cup (2, infty)
end{aligned}
$$

We know, that $ a_n=1$, so we know, that $ x_1+x_2+....+x_n=-2$.
It's all I have though, any
suggestions, what can I do in next step?










share|cite|improve this question











$endgroup$




I have to prove, that polynomial:



$ x^n + a_{n-1}x^{n-1}+...+a_2x^2+a_1x+a_0=0$
, where $ a_i= [-1,0,1] $ and $ i=0,1,.....,n $

doesn't have any solutions in the set
$$
begin{aligned}
( -infty,-2) cup (2, infty)
end{aligned}
$$

We know, that $ a_n=1$, so we know, that $ x_1+x_2+....+x_n=-2$.
It's all I have though, any
suggestions, what can I do in next step?







proof-verification polynomials






share|cite|improve this question















share|cite|improve this question













share|cite|improve this question




share|cite|improve this question








edited Dec 2 '18 at 8:48







Kukoz

















asked Dec 2 '18 at 8:29









KukozKukoz

278




278








  • 1




    $begingroup$
    elcome to MSE. Your question is phrased as an isolated problem, without any further information or context. This does not match many users' quality standards, so it may attract downvotes, or be put on hold. To prevent that, please edit the question. This will help you recognise and resolve the issues. Concretely: please provide context, and include your work and thoughts on the problem. These changes can help in formulating more appropriate answers.
    $endgroup$
    – José Carlos Santos
    Dec 2 '18 at 8:31










  • $begingroup$
    "elcome?" 😁😁😁
    $endgroup$
    – Robert Lewis
    Dec 2 '18 at 22:44














  • 1




    $begingroup$
    elcome to MSE. Your question is phrased as an isolated problem, without any further information or context. This does not match many users' quality standards, so it may attract downvotes, or be put on hold. To prevent that, please edit the question. This will help you recognise and resolve the issues. Concretely: please provide context, and include your work and thoughts on the problem. These changes can help in formulating more appropriate answers.
    $endgroup$
    – José Carlos Santos
    Dec 2 '18 at 8:31










  • $begingroup$
    "elcome?" 😁😁😁
    $endgroup$
    – Robert Lewis
    Dec 2 '18 at 22:44








1




1




$begingroup$
elcome to MSE. Your question is phrased as an isolated problem, without any further information or context. This does not match many users' quality standards, so it may attract downvotes, or be put on hold. To prevent that, please edit the question. This will help you recognise and resolve the issues. Concretely: please provide context, and include your work and thoughts on the problem. These changes can help in formulating more appropriate answers.
$endgroup$
– José Carlos Santos
Dec 2 '18 at 8:31




$begingroup$
elcome to MSE. Your question is phrased as an isolated problem, without any further information or context. This does not match many users' quality standards, so it may attract downvotes, or be put on hold. To prevent that, please edit the question. This will help you recognise and resolve the issues. Concretely: please provide context, and include your work and thoughts on the problem. These changes can help in formulating more appropriate answers.
$endgroup$
– José Carlos Santos
Dec 2 '18 at 8:31












$begingroup$
"elcome?" 😁😁😁
$endgroup$
– Robert Lewis
Dec 2 '18 at 22:44




$begingroup$
"elcome?" 😁😁😁
$endgroup$
– Robert Lewis
Dec 2 '18 at 22:44










1 Answer
1






active

oldest

votes


















1












$begingroup$

Let $f(x)=0$, then
begin{align}
0=&|x^n+a_{n-1}x^{n-1}+cdots+a_1x+a_0|\
geq&|x|^n-|a_{n-1}||x|^{n-1}-cdots-|a_1||x^1|-|a_0|
end{align}

Write it as
begin{align}
|x|^nleq & |a_{n-1}||x|^{n-1}+cdots+|a_1||x^1|+|a_0|\
leq & |x|^{n-1}+|x|^{n-2}+cdots+|x|+1\
= & frac{|x|^{n}-1}{|x|-1}
end{align}

If $|x|>2$, we get $|x|-1>1$ and then
$$
|x|^nleq |x|^n-1.
$$

It is impossible.






share|cite|improve this answer









$endgroup$













  • $begingroup$
    Could you tell me, why did you change $|a_0|$ to $1$?
    $endgroup$
    – Kukoz
    Dec 2 '18 at 13:24






  • 1




    $begingroup$
    @Kukoz Because $a_0in{-1,0,1}$, hence $|a_0|leq 1$.
    $endgroup$
    – yahoo
    Dec 2 '18 at 13:53













Your Answer





StackExchange.ifUsing("editor", function () {
return StackExchange.using("mathjaxEditing", function () {
StackExchange.MarkdownEditor.creationCallbacks.add(function (editor, postfix) {
StackExchange.mathjaxEditing.prepareWmdForMathJax(editor, postfix, [["$", "$"], ["\\(","\\)"]]);
});
});
}, "mathjax-editing");

StackExchange.ready(function() {
var channelOptions = {
tags: "".split(" "),
id: "69"
};
initTagRenderer("".split(" "), "".split(" "), channelOptions);

StackExchange.using("externalEditor", function() {
// Have to fire editor after snippets, if snippets enabled
if (StackExchange.settings.snippets.snippetsEnabled) {
StackExchange.using("snippets", function() {
createEditor();
});
}
else {
createEditor();
}
});

function createEditor() {
StackExchange.prepareEditor({
heartbeatType: 'answer',
autoActivateHeartbeat: false,
convertImagesToLinks: true,
noModals: true,
showLowRepImageUploadWarning: true,
reputationToPostImages: 10,
bindNavPrevention: true,
postfix: "",
imageUploader: {
brandingHtml: "Powered by u003ca class="icon-imgur-white" href="https://imgur.com/"u003eu003c/au003e",
contentPolicyHtml: "User contributions licensed under u003ca href="https://creativecommons.org/licenses/by-sa/3.0/"u003ecc by-sa 3.0 with attribution requiredu003c/au003e u003ca href="https://stackoverflow.com/legal/content-policy"u003e(content policy)u003c/au003e",
allowUrls: true
},
noCode: true, onDemand: true,
discardSelector: ".discard-answer"
,immediatelyShowMarkdownHelp:true
});


}
});














draft saved

draft discarded


















StackExchange.ready(
function () {
StackExchange.openid.initPostLogin('.new-post-login', 'https%3a%2f%2fmath.stackexchange.com%2fquestions%2f3022397%2fpolynomial-of-the-nth-degree-and-its-elements%23new-answer', 'question_page');
}
);

Post as a guest















Required, but never shown

























1 Answer
1






active

oldest

votes








1 Answer
1






active

oldest

votes









active

oldest

votes






active

oldest

votes









1












$begingroup$

Let $f(x)=0$, then
begin{align}
0=&|x^n+a_{n-1}x^{n-1}+cdots+a_1x+a_0|\
geq&|x|^n-|a_{n-1}||x|^{n-1}-cdots-|a_1||x^1|-|a_0|
end{align}

Write it as
begin{align}
|x|^nleq & |a_{n-1}||x|^{n-1}+cdots+|a_1||x^1|+|a_0|\
leq & |x|^{n-1}+|x|^{n-2}+cdots+|x|+1\
= & frac{|x|^{n}-1}{|x|-1}
end{align}

If $|x|>2$, we get $|x|-1>1$ and then
$$
|x|^nleq |x|^n-1.
$$

It is impossible.






share|cite|improve this answer









$endgroup$













  • $begingroup$
    Could you tell me, why did you change $|a_0|$ to $1$?
    $endgroup$
    – Kukoz
    Dec 2 '18 at 13:24






  • 1




    $begingroup$
    @Kukoz Because $a_0in{-1,0,1}$, hence $|a_0|leq 1$.
    $endgroup$
    – yahoo
    Dec 2 '18 at 13:53


















1












$begingroup$

Let $f(x)=0$, then
begin{align}
0=&|x^n+a_{n-1}x^{n-1}+cdots+a_1x+a_0|\
geq&|x|^n-|a_{n-1}||x|^{n-1}-cdots-|a_1||x^1|-|a_0|
end{align}

Write it as
begin{align}
|x|^nleq & |a_{n-1}||x|^{n-1}+cdots+|a_1||x^1|+|a_0|\
leq & |x|^{n-1}+|x|^{n-2}+cdots+|x|+1\
= & frac{|x|^{n}-1}{|x|-1}
end{align}

If $|x|>2$, we get $|x|-1>1$ and then
$$
|x|^nleq |x|^n-1.
$$

It is impossible.






share|cite|improve this answer









$endgroup$













  • $begingroup$
    Could you tell me, why did you change $|a_0|$ to $1$?
    $endgroup$
    – Kukoz
    Dec 2 '18 at 13:24






  • 1




    $begingroup$
    @Kukoz Because $a_0in{-1,0,1}$, hence $|a_0|leq 1$.
    $endgroup$
    – yahoo
    Dec 2 '18 at 13:53
















1












1








1





$begingroup$

Let $f(x)=0$, then
begin{align}
0=&|x^n+a_{n-1}x^{n-1}+cdots+a_1x+a_0|\
geq&|x|^n-|a_{n-1}||x|^{n-1}-cdots-|a_1||x^1|-|a_0|
end{align}

Write it as
begin{align}
|x|^nleq & |a_{n-1}||x|^{n-1}+cdots+|a_1||x^1|+|a_0|\
leq & |x|^{n-1}+|x|^{n-2}+cdots+|x|+1\
= & frac{|x|^{n}-1}{|x|-1}
end{align}

If $|x|>2$, we get $|x|-1>1$ and then
$$
|x|^nleq |x|^n-1.
$$

It is impossible.






share|cite|improve this answer









$endgroup$



Let $f(x)=0$, then
begin{align}
0=&|x^n+a_{n-1}x^{n-1}+cdots+a_1x+a_0|\
geq&|x|^n-|a_{n-1}||x|^{n-1}-cdots-|a_1||x^1|-|a_0|
end{align}

Write it as
begin{align}
|x|^nleq & |a_{n-1}||x|^{n-1}+cdots+|a_1||x^1|+|a_0|\
leq & |x|^{n-1}+|x|^{n-2}+cdots+|x|+1\
= & frac{|x|^{n}-1}{|x|-1}
end{align}

If $|x|>2$, we get $|x|-1>1$ and then
$$
|x|^nleq |x|^n-1.
$$

It is impossible.







share|cite|improve this answer












share|cite|improve this answer



share|cite|improve this answer










answered Dec 2 '18 at 8:55









yahooyahoo

606412




606412












  • $begingroup$
    Could you tell me, why did you change $|a_0|$ to $1$?
    $endgroup$
    – Kukoz
    Dec 2 '18 at 13:24






  • 1




    $begingroup$
    @Kukoz Because $a_0in{-1,0,1}$, hence $|a_0|leq 1$.
    $endgroup$
    – yahoo
    Dec 2 '18 at 13:53




















  • $begingroup$
    Could you tell me, why did you change $|a_0|$ to $1$?
    $endgroup$
    – Kukoz
    Dec 2 '18 at 13:24






  • 1




    $begingroup$
    @Kukoz Because $a_0in{-1,0,1}$, hence $|a_0|leq 1$.
    $endgroup$
    – yahoo
    Dec 2 '18 at 13:53


















$begingroup$
Could you tell me, why did you change $|a_0|$ to $1$?
$endgroup$
– Kukoz
Dec 2 '18 at 13:24




$begingroup$
Could you tell me, why did you change $|a_0|$ to $1$?
$endgroup$
– Kukoz
Dec 2 '18 at 13:24




1




1




$begingroup$
@Kukoz Because $a_0in{-1,0,1}$, hence $|a_0|leq 1$.
$endgroup$
– yahoo
Dec 2 '18 at 13:53






$begingroup$
@Kukoz Because $a_0in{-1,0,1}$, hence $|a_0|leq 1$.
$endgroup$
– yahoo
Dec 2 '18 at 13:53




















draft saved

draft discarded




















































Thanks for contributing an answer to Mathematics Stack Exchange!


  • Please be sure to answer the question. Provide details and share your research!

But avoid



  • Asking for help, clarification, or responding to other answers.

  • Making statements based on opinion; back them up with references or personal experience.


Use MathJax to format equations. MathJax reference.


To learn more, see our tips on writing great answers.




draft saved


draft discarded














StackExchange.ready(
function () {
StackExchange.openid.initPostLogin('.new-post-login', 'https%3a%2f%2fmath.stackexchange.com%2fquestions%2f3022397%2fpolynomial-of-the-nth-degree-and-its-elements%23new-answer', 'question_page');
}
);

Post as a guest















Required, but never shown





















































Required, but never shown














Required, but never shown












Required, but never shown







Required, but never shown

































Required, but never shown














Required, but never shown












Required, but never shown







Required, but never shown







Popular posts from this blog

Plaza Victoria

In PowerPoint, is there a keyboard shortcut for bulleted / numbered list?

How to put 3 figures in Latex with 2 figures side by side and 1 below these side by side images but in...